Create an equation that is parallel to the line y = 2x + 3 and goes through the point (0, -2).

Answers

Answer 1
0,-2 substitution to equation
-2=2(0)+3
-2=3
Not parallel

Related Questions

How can you rewrite -8 -10 as an addition problem? ​

Answers

Answer:

-8 + (-10)

Hope this help!

express y in terms of x: 6(x-y) = 19

Answers

Expressing y in terms of x gives y = x - 19/ 6

What is subject of formula

Subject of formula can be defined as the variable that is being expressed in terms of other variables in an equation, function or expression.

It is also described as the variable that is being worked out in an expression or equation.

It is made to stand alone on on side of the equality sign.

We have the equation;

6(x-y) = 19

To make 'y' the subject of formula, let's take the following steps;

Divide both sides by 6, we have;

x - y = 19/ 6

now, take the variable 'x' over the equality sign

-y = 19/ 6 - x

Divide both sides by -1

y = x - 19/ 6

Hence, the equation is y = x - 19/ 6

Learn more about subject of formula here:

https://brainly.com/question/10643782

#SPJ1

what is −2x+y=15? i cant undestand

Answers

Step-by-step explanation:

it is a linear equation

Normally two if them are given together to solve

hope it helps

If you partition a line segment in a ratio of 4:1, what is the fraction?

Answers

Answer:

Your answer would be 4 over 1

A particular bacterium population is found to double the in 20 minutes. If a laboratory culture
begins with a population of P=340 of this bacteria and there is no change in the growth rate, how
many bacteria will be present in t=37 minutes?
Hint: Use equation P = Poekt, where k is the relative growth. Round to two decimal places.

Answers

The bacteria present at t=37 minutes is found to be 8.40, with no change in growth rate.

What does population growth exponentially mean?

When a population's per capita growth rate remains constant, regardless of population size, exponential growth occurs, causing the population to grow exponentially as the population increases.

Given:

P = 340

It has been discovered that a specific bacterial population doubles in 20 minutes.

k = 2/20 = 0.1

t = 37 minutes

We know that,

P = P₀[tex]e^{kt}[/tex]

340   = P₀([tex]e^{0.1*37}[/tex])

340 =P₀(40.44)

P₀ = 8.40

As a result, the bacteria that will be present in t=37 minutes is found to be 8.40 with no change in growth rate.

Learn more about exponential growth here:

https://brainly.com/question/13223520

#SPJ9

Model with Mathematics
The owner of a frozen yogurt stand considers different pricing options for a serving of yogurt plus toppings. For what number of toppings do the two options give the same price?
A. Define the variables.
B. Write a system of equations that models this situation.
C. Graph the system. What is the intersection point?
D. What is the solution of the problem?
E. Is the solution reasonable for this situation? Explain.

Answers

Answer:

Step-by-step explanation:

A, B:  Define the Variables and Write Equations

Let T be the number of toppings.

Option 1:  $3 + $0.75*T

Option 2:  $4 + $0.50*T

C.  Graph the System

See the attached graph.  The intersection point is (4,6).  This means that 4 toppings will result in a cost of $6 for both yogurt options.

D.  Solution to the Problem

The solution is given above, but we can also solve using equations, instead of graphing.  We want the value of T that will make these expressions equal:

$3 + $0.75*T = $4 + $0.50*T

$0.25*T = $1

T = 4 toppings

=============

CHECK:

For 4 toppings:

Option 1:  $3 + $0.75*T

                  $3 + $0.75*(4)

                  $3 + $3 = $6

Option 2:  $4 + $0.50*T

                  $4 + $0.50*(4)

                  $4 + $2 = $6

Four toppings will make both options equal in cost.

E:  Is the solution reasonable?

Yes.  The cheaper option ($3) has a more expensive topping, so at some point (at 4 toppings) it will be the same price as the second option, with a higher initial price, but with cheaper toppings.

Can someone help with questions 3,4 and 5? THANK YOUUU ^^ I did questions 1 and 2

Answers

Answer:

Question 3.

[tex]y = \frac{2}{7} x - 3[/tex]

question 4

[tex]y = - 4x - 1[/tex]

question 5

[tex]y = 5x + 3[/tex]

Step-by-step explanation:

Question 3

1.Write down what is given in equation form

[tex]y = \frac{2}{7} x + c[/tex]

2. Insert the given point to work out c

[tex] - 5 = \frac{2}{7} ( - 7) + c[/tex]

[tex]c = - 3[/tex]

3. answer

[tex]y = \frac{2}{7} x - 3[/tex]

Question 4

1 write what was given in an equation

[tex]y = - 4x + c[/tex]

2. insert given point (-1;3) to work out c

[tex]3 = - 4( - 1) + c[/tex]

[tex]c = - 1[/tex]

3. final answer

[tex]y = - 4x - 1[/tex]

question 5

1.with what was given write it in equation form

[tex]y = 5x + c[/tex]

2. substitute the given point (-2;-7) to work out c

[tex] - 7 = 5( - 2) + c[/tex]

[tex]c = 3[/tex]

3.final answer

[tex]y = 5x + 3[/tex]


For the given function, find (a) f(1) (b) f(-2).
(a) f(1) =
(b) f(-2) =

Answers

For the given functions , f(1) = -3 and f(-2) = -6 .

In the question

a graph is given

we have to find the value of the functions  f(1) and f(-2)

the value of f(1) means the value of y, when x=1 .

we see that ,

on the graph when x = 1 , the value of y = -3 .

so the value of [tex]f(1)[/tex] = -3 .

the value of f(-2) means the value of y, when x = -2 .

we see that ,

on the graph when x = -2 , the value of y = -6 .

so the value of [tex]f(-2)[/tex] = -6 .

Therefore , For the given functions , f(1) = -3  and f(-2) = -6 .

Learn more about Functions here

https://brainly.com/question/18240375

#SPJ1

Why do visit the zoo on cold days

Answers

Answer:

How is this math?

Step-by-step explanation:

Answer:

Visiting the zoo on cold days limits the number of people there and it isn't too hot so you don't stand in the sweltering heat. Besides, on cold days, the animals may be in a better mood, but be cautioned, remember to bring a jacket just in case.

Step-by-step explanation:

Three consecutive intergers are such that four times the least integer is three times the greatest . What is the greatest of this integers? A-16, B-8, C-10, D-12, E-14

Answers

If we call the middle number in the sequence [tex]x[/tex], the numerical values of the other terms will be [tex]x-1[/tex] and [tex]x+1[/tex], respectively, since they are consecutive.

We are told that [tex]4[/tex] times the smallest number is equal to [tex]3[/tex] times the largest number. Let us write our equality.

[tex]4(x-1)=3(x+1)[/tex][tex]4x-4=3x+3[/tex][tex]4x-3x=4+3[/tex][tex]x=7[/tex]

We found our number in the middle. Since it wants the value of our largest number, let's add 1 to the middle number.

[tex]x+1=7+1=8[/tex]

The answer is B.

You have a $25 calling card. calls made using the card within the united states costs $10 per minute. while calls made from the us to france cost $25 per minute. write an inequality that relates the number of minutes x you can use for calls within the us and the number of minutes y you can use from de us and france

Answers

Answer:

10× + 25y ≤ 25

Answer:

0.1x + 0.25y ≤ 25

Step-by-step explanation:

A call within the US is $0.10, a call to France is $0.25, and you have $25.00. Your total amount of money spent depends on how many calls you make. Now, turn this information into an inequality.

10. (2, -4); slope = -3

Answers

The equation (-4 = -3x + 2) is formed and plotted on the graph (Refer to the graph attached below).

What is a graph?In mathematics, a graph is a visual representation or diagram that shows facts or values in an ordered way. The relationships between two or more items are frequently represented by the points on a graph. Charts and graphs come in a variety of styles. Line graphs, bar graphs and histograms, pie charts, and Cartesian graphs are likely the four most popular types.

So, the equation can be:

The slope is -3 and the coordinates are (2, -4):Formula: y = MX + bWhere m is the slope.

From the equation:

y = MX + b-4 = -3x + 2

Now, plot the equation on the graph as follows:

(Refer to the graph attached below)

Therefore, the equation (-4 = -3x + 2) is formed and plotted on the graph (Refer to the graph attached below).

Know more about graphs here:

https://brainly.com/question/19040584

#SPJ13

The correct question is given below:
From the equation and plot it on a graph when, (2, -4) and slope = -3.

Four students are sharing 1/3 carton of yogurt
equally. How much yogurt will each student
get

Answers

Answer: 1/12

Step-by-step explanation:

1) Set up the equation

1/3 / 4

2) Change 4 into a fraction

1/3 / 4/1

3) Change the divide into a multiply by flipping the 2nd fraction

1/3 * 1/4

4) Solve

1/12

FRUIT Ronald buys fresh fruit from a fruit stand. Apples cost $5 per pound and peaches cost $6 per pound. He has $60 to spend. The table
shows the function relating the number of pounds of apples, x, and the number of pounds of peaches, y, Ronald could purchase.
Apples (lb) Peaches (lb)
X
y
0
1.2
3
6
7.2
12
The ?
y
10
9
7.5
5
4
0
Hintercept shows that if Ronald buys only apples, he can buy ?
intercept shows that if Ronald buys only peaches, he can buy ?
pounds. The
pounds

Answers

Answer:

a

Step-by-step explanation:

your a could be the same.

Sixty and sixty eight thousandths

Answers

Answer: 60 and 68/1000

Step-by-step explanation:

Not sure if you wanted it in fraction, decimal, or percentage form.

Answer:

60 and 68.000

Step-by-step explanation:

i not sure if that was what u were looking for

The perimeter of AMUD is 38cm.
m/D=
MD=
14cm
360
Blank 1:
Blank 2:
cm
M
D

Answers

The side MD measures 10 cm and ∠D=72° for the given triangle.

What is triangle?

A shape with corners and three vertices is called a triangle. It is one of the most fundamental geometric shapes. Triangle ABC is the designation for a triangle containing vertices A, B, & C. In Euclidean geometry, all three points that are not collinear produce a singular triangle and a singular plane.

The perimeter of ΔMUD= 38cm

MU=DU=14cm

∠U=36°

⇒∠M=∠D=x

Applying Angle sum property,

36°+x+x=180°

2x=180°-36°

x=72°

∴∠D=72°

Perimeter of triangle=14+14+MD

38=28+MD

⇒MD=10cm.

The side MD measures 10 cm and ∠D=72° for the given triangle.

To learn more about triangles visit:

https://brainly.com/question/25215131

#SPJ1

Manvita deposits Rs. 5000 in her bank account after two days. She withdraws Rs.
3748 from it. If the amount deposited is a positive integer. How will you represent
the amount withdrawn and also find the balance amount in the account?

Answers

If the amount deposited is a positive integer , so the amount withdrawn is represented by -3748  and the balance amount in the Manvita's account is Rs 1252 .

In the question ,

it is given that

Money deposited by Manvita =  Rs 5000 .

Money withdrawn by Manvita = Rs 3748 .

Since , amount deposited is a positive integer ,

So, the amount withdrawn will be a negative integer

Balance in Manvita's account = Money deposited + Money Withdrawn

Substituting the values , we get

Balance in Manvita's account = 5000 + (-3748)

= 5000 - 3748

= 1252

Therefore , if the amount deposited is a positive integer , so the amount withdrawn is represented by -3748  and the balance amount in the Manvita's account is Rs 1252 .

Learn more about Equation here

https://brainly.com/question/17463944

#SPJ1

with show work. trigonometry

Answers

60.8. you would have to combine all know angles and subtract them by 180

What is -3 times 4??

Answers

Answer:

-12

Step-by-step explanation:

hope it helps

have a nice evening

Which relation represents a function?

Answers

A and B represent a function

Write an equation point-slope form of the line that passes through the point (3,3) with slope 5/3

Answers

We solve as follows:

[tex]y-3=\frac{5}{3}(x-3)[/tex][tex]\Rightarrow y-3=\frac{5}{3}x-5\Rightarrow y=[/tex]

Monique built a wooden frame woth dimensions 0.8m by0.5m.how much wood will be left from a 3.4m length of timber

Answers

Answer: 0.8 m

Step-by-step explanation:

1.  The perimeter P of the wooden frame is equal to:

P=2(0.8+0.5)

P=2(1.3)

P=2.6 m

2. The remaining length of wood is equal to:

3.4-2.6=

0.8 m

The following figure shows the entire graph of a relationship.
Does the graph represent a function?
Yes or no​

Answers

Answer:

No

Step-by-step explanation:

The graph fails the vertical line test. More than one y-value is on one x-value.

3×(9-4)³ - 46×3

A) 1
B) 237
C) 711
D) 987

Answers

Answer:

237

Step-by-step explanation:

3×(9-4)³ - 46×3
subtract 4 from 9 to get 5
3x5³-46x3
calculate 5 to the power of 3 and get 125
3x125-46x3

multiply 3 and 125 to get 375.

375-46x3
Multiply 46 and 3 to get 138
375-138

subtract 138 from 375 to get 237.
And there's your answer

You can follow the rule of BODMAS.

BODMAS stands for Bracket, Of, Division, Multiplication, Addition, Subtraction

By following the rule of BODMAS, the solution will be like this:

3×(5)³ - 46×3

= 3×125 - 46×3

= 375 - 138

= 237

I hope my explanation has helped you.

You can do well to declare this answer as the Brainiest Answer.

Thanks so much

A group of 40 children attended a baseball game on a field trip. Each child received either a hot dog or a bag of popcorn. Hot dogs were $2.25 and popcorn was $1.75. If the total bill was $83.50, how many hotdogs and bags of popcorn were purchased?

Write the equations for the situation above. (Use the variable x for hotdogs and y for popcorn)

Answers

Answer:

See Explanation

Step-by-step explanation:

[tex]x+y=40\\2.25x+1.75y=83.5\\[/tex]

from the first you have

[tex]h=40-p[/tex] substitute into the second:

[tex]2.25(40-p)+ 1.75x= 83.5\\90-2.25y+1.75x= 83.5[/tex]remaining

[tex]0.5y=6.5\\p= \frac{6.5}{0.5} =13\\[/tex]

substitute back into [tex]h= 40 - y\\h = 40 - 13 = 27[/tex]

Hope this helped :)

what is the highest in order from least to greatest

Answers

Answer:

Least to greatest: 1. International finance center

2. Petronas towers

3. Zifeng towers

4. Burj Khalifa

PLEASE SOMEONE HELP ME
question 6
The exponential function in the graph shows the value, in dollars, of an investment over time. What real-world information is given by each feature of the graph?

THE TWO PICTURES GOES WITH QUESTION 6



Question 12
Evan runs a website that has gotten quite popular. However, its popularity seems to be decreasing and the site is now getting 7% fewer hits each week. If Evan's site got 34,000 hits this week, how many weekly hits can he expect it to get 3 weeks from now?

Round to the nearest whole number.
A. 134,360 hits
B. 27,348 hits
C. 41,651 hits
D. 714,000 hits

Answers

Considering exponential functions, it is found that:

6.

(1.7.5): The value after one year.y-intercept of 10: starting value.

12. The number of hits 3 weeks from now will be of 27,348, hence option B is correct.

What is an exponential function?

An exponential function is modeled according to the rule given next:

[tex]y = ab^x[/tex]

The parameters of the exponential function are defined as follows:

a is the y-intercept, which is the numeric value of the function when x = 0.b is the rate of change of the function.

For item 6, considering the points (0,10) and (1,7.5), the parameters are given as follows:

y-intercept of 10, which is the starting value.When x = 1, y = 7.5, given the value after one year.

For item 12, the initial amount and the rate of change are given as follows:

a = 34,000, b = 1 - 0.07 = 0.93.

Hence the number of hits x weeks from now is modeled according to the following function:

[tex]h(x) = 34000(0.93)^x[/tex]

After 3 weeks, x = 3, hence the number of hits will be of:

[tex]h(3) = 34000(0.93)^3 = 27348[/tex]

Meaning that option B is correct.

More can be learned about exponential functions at https://brainly.com/question/25537936

#SPJ1

Determine the y-value of each ordered pair based on the given x-value
y= -x + 9; (-9, ), (1, ), (5, )

Brainfiy to correct person

Answers

The value of y in each case are 18,8 and 4 respectively.

Given equation:-

y = -x + 9

Values of x = {-9, 1, 5}

We have to find the corresponding values of y for each value of x.

Putting x = -9 in the given equation, we get,

y = -(-9) + 9 = 9 + 9 = 18

Putting x = 1 in the given equation, we get,

y = -1 + 9 = 8

Putting x = 5 in the given equation, we get,

y = -5 + 9 = 4

Hence, the values of y are {18, 8,4}

The complete ordered pair of (x,y) is given by:-

(-9,18)

(1,8)

(5,4)

To learn more about ordered pair, here:-

https://brainly.com/question/28874341

#SPJ1

What is the answer to this
|-0.4| ?

Answers

Answer:

0.4

Step-by-step explanation:

The absolute value is just the number in the line things but positive.

The absolute value is the distance between a number and zero. The distance between − 0.4 and 0 is 0.4

if |-0.4| is all that is in the question it is 0.4

If pH = -log [H+], find the pH of a substance that has a hydrogen concentration [H+] = 5.7 X 10-4.Round your answer to the nearest ten thousandth (4 decimals).

Answers

[H+] = -1.7243

Explanation:

Since [H+] = 5.7 * 10 - 4, in the function -log[H+] representing pH, replace H+

pH = -log [5.7 *10 - 4]

pH = -log[53]

pH = -1.7243

NB:

This number is rounded up from -1.72427.....

Other Questions
What is the value of x in the geometric sequence x, 3, .A. -27B. -9C. -1D. -3 A trapezoid has bases 4 and 12 and area of 40m^2 find the height peng company is considering buying a machine that will yield income of $1,950 and net cash flow of $14,950 per year for three years. the machine costs $45,000 and has an estimated $6,000 salvage value. compute the accounting rate of return for this investment. The text states, "The Maya found clever ways to meet thechallenges of their diverse landscapes." Select an example thatsupports the statement.O In dry areas, the Maya built aqueducts to bring water from otherregions.O In lowland areas, the Maya created raised-earth platforms and canalsto redirect and reuse the water.O The Maya used quetzal feathers to decorate headdresses.O The rain and mud of the rainforest presented many challenges. A straight road makes an angle of 15 degrees with the horizontal. When the angle of elevation of the sun is 57 degrees, a vertical pole at the side of the road casts a shadow 75 feet long directly down the road, as shown in the figure. Approximate the length of the pole. Round to the nearest hundredth. during the winter olympics the us curling team scored 2 points more than the canadian team. the canadian curling team scored 4 points more than the norwegian team. altogether the three teams scored a total of 208 point. how many points did the canadian team scored? HELLP FOR 20 PTS PLEASEEE Pls I need the answer quick The average number of babies born at a private hospital's maternity wing is 6 per hour. What is the probability that three babies are born during a particular 1-hour period in this maternity wing? The management of the UNICO department store has decided to enclose a 903 ft2 area outside the building for displaying potted plants and flowers. One side will be formed by the external wall of the store, two sides will be constructed of pine boards, and the fourth side will be made of galvanized steel fencing. If the pine board fencing costs $7/running foot and the steel fencing costs $4/running foot, determine the dimensions of the enclosure that can be erected at minimum cost. During an annual check-up, the doctor typically feels the lymph nodes for swelling. What would swelling in the lymph nodes most likely indicate? A. the body is experiencing an infection B. white blood cells are lacking nutrients C. white blood cells are growing uncontrollably D. the body is not receiving enough oxygen A car journey is m miles long.One kilometre is equivalent to x miles.The car uses one litre of fuel to travel a distance of f kilometres.Fuel for the car costs p pence per litre.Which of the following expressions gives the cost of fuel for this journey, in pounds? Simplify the complex rational expression. When typing your answer for the numerator and denominator be sure to type the term with the variable first with no spaces between characters. If an exponent is needed use the carrot key (^) by pressing shift and 6 at the same time.\frac{\left(\frac{2}{a}+\frac{7}{b}\right)}{b}The numerator is AnswerThe denominator is Answer What value of x satisfies the equation 4x + 2 = 22?A 3.5B 5.0C 6.0D 7.5 A car of mass m = 1090 kg is traveling down a = 11 degree incline. When the car's speed is v0 = 16 m/s, a mechanical failure causes all four of its brakes to lock. The coefficient of kinetic friction between the tires and road is k = 0.45.Calculate the distance the car travels down the hill L in meters until it comes to a stop at the end. socioemotional changes that affect parent-adolescent relationships include multiple choice increased responsibility in school. increased initiative in school. development of more sophisticated friendships. all of these choices are correct. my daughter is working on expression problems. i cant remember how to do this...the problem is 12(3y+x)please help me tell her how to write and solve this. Is this a run-on sentence?Before the start of the Revolutionary War, the British thought it would be easy to defeat the colonists, whereas Britain had an army and the world's most powerful navy, the colonists had neither. A certain red light has a wavelength of 680 nm.a. What is the frequency of the light? I need asap answers for this question. Im stuck.